LSAT and Law School Admissions Forum

Get expert LSAT preparation and law school admissions advice from PowerScore Test Preparation.

 Clay Cooper
PowerScore Staff
  • PowerScore Staff
  • Posts: 241
  • Joined: Jul 03, 2015
|
#30076
Hi Rita,

Thanks for your question.

"Researchers found no correlation..." does not mean there is no correlation; no one here is claiming to have proved anything. All that the correct answer choice asserts is that these findings suggest that the correlation might in fact not exist. The research certainly does seem to suggest that; there is no need for us to assume that the research proves it (that would be, as you point out, a bad assumption).

Hope that helps!
 AAron24!
  • Posts: 12
  • Joined: Aug 09, 2020
|
#78722
`Hi Powerscore,
Out of curioisty, if there was an answer choice that something along the lines of, "The study shows that people that everyone has their own experiences with arthirits." Wouldnt this also be an accurate completion of the argument as it shows that everyone that has arthriits also has different responses to various weathers? This was my prephrase to the question.
 Robert Carroll
PowerScore Staff
  • PowerScore Staff
  • Posts: 1787
  • Joined: Dec 06, 2013
|
#81127
Aaron,

There are two aspects to your prephrase that I think don't quite hit the mark. First, note that the correct answer says the study "suggests" something. I think that's actually a good thing for an answer to say - we have one study, and shouldn't be too quick to make sweeping conclusions from it. So saying the study "suggests" rather than "proves" or "shows" is important.

Further, the information about the study we have is focused on correlations (or lack thereof!) between arthritic pain and weather conditions. A prephrase that generalizes beyond that situation to "experiences" more generally is not limited to pain and weather like it should be.

Robert Carroll
User avatar
 ange.li6778
  • Posts: 34
  • Joined: Dec 27, 2021
|
#96475
Hi Powerscore, I'd really like some additional clarification on this question. Is B wrong because the stimulus says the sufferers think weather factors caused increased pain while B is just about causes of pain? So the sufferers aren't saying they believe weather factors caused their pain but rather made it worse? Otherwise, it seems to me that B is vague enough to also mean that the sufferers are imagining their increased pain when there's weather factors. I can usually reason out why the right answer is right but this time I'm really stumped. Thanks in advance!
User avatar
 Henry Z
  • Posts: 61
  • Joined: Apr 16, 2022
|
#96680
I see the two main reasons (B) is wrong are 1) without support, it says beliefs cause pain, and 2) "indicate" is too strong.

But 1) does "affect" suggest a causal relationship? I understand it just means "change" (and not even always change), so the beliefs don't cause the pain, but only changes one's "assessment" of the pain. For instance, the belief about the cause of the pain may make the arthritis sufferer more conscious of a dull pain when "the relevant feature of the weather" occurs. Sufferers giving varying accounts may be because of their varying extents of the beliefs, and sometimes the beliefs don't affect at all (note the stimulus says "MAY affect"). The real cause may be neither weather nor the belief.

2) "indicate" in (B) may be stronger than "suggest" in (C), but doesn't the aforementioned "MAY", which means may or may not, soften the language enough?
 Adam Tyson
PowerScore Staff
  • PowerScore Staff
  • Posts: 5153
  • Joined: Apr 14, 2011
|
#97233
"Affect" is a causal idea, Henry. The problem with answer B is that belief isn't tied to the INTENSITY of the pain, but to the CAUSE of the pain. It's not about how much pain they are in, but what is causing that pain in the first place.

"May" does soften that language, as you say, but the answer is still about the wrong thing being indicated. The study tells us something about the cause of the pain, not about the intensity of that pain.
User avatar
 ashpine17
  • Posts: 321
  • Joined: Apr 06, 2021
|
#99339
wait, so was the initial post saying that B was wrong because it was factually NOT supported or it was wrong because the emphasis was wrong
User avatar
 ashpine17
  • Posts: 321
  • Joined: Apr 06, 2021
|
#99340
when reading the stimulus, am i supposed to side more with the researchers in terms of who is correct about the causes of the pain?
 Adam Tyson
PowerScore Staff
  • PowerScore Staff
  • Posts: 5153
  • Joined: Apr 14, 2011
|
#99375
You aren't really supposed to take a side, ashpine, but you are supposed to complete the argument, so you should put yourself in the position of the author of the argument. What would they say about what the study shows? Not the researchers, and not the arthritis sufferers, but the author of the argument. Complete their argument for them. Supply their conclusion, based on what they said about the results of the study.

And yes, there is no factual basis in the argument to support answer B, since the study gave us no information about the degree to which they accurately assessed the intensity of their pain. The may have been 100% accurate about the pain intensity, but still completely imagined that the weather had something to do with it.

Get the most out of your LSAT Prep Plus subscription.

Analyze and track your performance with our Testing and Analytics Package.